LSAT and Law School Admissions Forum

Get expert LSAT preparation and law school admissions advice from PowerScore Test Preparation.

 est15
  • Posts: 94
  • Joined: Aug 28, 2013
|
#15985
Hi, I'm not understanding why (D) is incorrect. I negated (D) and thought that hurt the argument because if improvements in doctors' use of techniques of curative medicine resulted in decreases of overall medical costs, that would make medicine more cost-effective anyway and thus medical schools won't need to spend more time teaching preventative medicine.
 Ron Gore
PowerScore Staff
  • PowerScore Staff
  • Posts: 220
  • Joined: May 15, 2013
|
#16008
Hi Est,

Remember that when you use the Assumption Negation Technique, you are taking the logical opposite of the answer choice, not the polar opposite. Your statement, that it would result in decreases, indicates that you took the polar opposite of "only increase overall medicinal costs." The logical opposite of that phrase would be "not only increase overall medicinal costs," which would include the possibility that they would stay the same.

Also, this answer choice doesn't really get to the point of the argument, which is about the benefit of preventative medicine and the benefit of teaching it. This answer choice addresses the wrong portion of the argument.

Please let me know if I can help further!

Ron
 reop6780
  • Posts: 265
  • Joined: Jul 27, 2013
|
#16177
The correct answer is E while I chose A.

The conclusion is that when the goal is cost-effectiveness of medicine, medical schools spend insufficient time teaching preventive medicine.

I don't understand how "time required" to teach preventive medicine comes into playing in the answer.

Depending upon goals, "time required" for preventive medicine class can differ.

The conclusion is conditional: "if their goal is to make medicine more cost-effective..."

I don't see how the negated answer E weakens the conclusion either.

The time required can be less or same as what is spent now, and it does not seem to do anything for the conclusion from my understanding. :cry:
 David Boyle
PowerScore Staff
  • PowerScore Staff
  • Posts: 836
  • Joined: Jun 07, 2013
|
#16200
reop6780 wrote:The correct answer is E while I chose A.

The conclusion is that when the goal is cost-effectiveness of medicine, medical schools spend insufficient time teaching preventive medicine.

I don't understand how "time required" to teach preventive medicine comes into playing in the answer.

Depending upon goals, "time required" for preventive medicine class can differ.

The conclusion is conditional: "if their goal is to make medicine more cost-effective..."

I don't see how the negated answer E weakens the conclusion either.

The time required can be less or same as what is spent now, and it does not seem to do anything for the conclusion from my understanding. :cry:
Hello,

The stimulus says, teach more preventative medicine.
But that assumes, as per answer E, that more prev. med. needs to be taught. Maybe they're teaching all they need to, even in the 1-10 ratio.
The negated version of answer E basically says that more prev. med. doesn't need to be taught, which destroys the argument.

David
 srcline@noctrl.edu
  • Posts: 243
  • Joined: Oct 16, 2015
|
#21972
Hello, David

I am not understanding this question either. I was in between answer choices A and E. This is what I got for the negation of A and E.

Conclusion: If their goal is to make medicine more cost effective, medical schools spend insufficient time teaching preventive medicine.

A; Preventive medicine makes use of technologies that are not lower in initial cost than the technologies used within the practice if curative medicine.

E: The time required to teach preventive medicine throughly is not greater than one hour for every ten that are now spent teaching curative medicine.

Is A not correct because technologies is a new element not mentioned in the stimulus and therefore does not weaken the conclusion?

Thankyou
Sarah
 Emily Haney-Caron
PowerScore Staff
  • PowerScore Staff
  • Posts: 577
  • Joined: Jan 12, 2012
|
#21973
Hi Sarah,

Here, the costs of the technologies is irrelevant - it has nothing to do with the stimulus. We KNOW using techniques of preventative medicine "cuts down medical costs greatly." How the cost of technology plays into that doesn't matter at all.

But what we don't know is how much time med students need to spend learning preventative medicine in order to see that decrease in cost. Maybe there is just a little bit to learn, and spending that one hour for every 10 is maybe sufficient, and spending additional time would not cut costs any extra. Or maybe, there is tons to learn, and spending much more time on preventative medicine would lead to even greater reductions in costs. We don't know. The assumption that is being made, therefore, is E - that more time is needed than is currently being spent (thus the conclusion that more time would make medicine more cost effective).

Hope that helps!
 lsatstudying11
  • Posts: 54
  • Joined: Jul 30, 2020
|
#78089
Hi,

I see how E is correct! But I was wondering if another underlying assumption here is that increasing the number of hours dedicated to teaching preventative medicine will not (in net terms) drastically reduce cost-efficiency by taking away the number of hours dedicated to curative medicine, which might also help to reduce costs (and might also even be better than preventative medicine in cutting costs?)? Thank you! :-D
 Rachael Wilkenfeld
PowerScore Staff
  • PowerScore Staff
  • Posts: 1358
  • Joined: Dec 15, 2011
|
#81066
Hi studying,

Often in these sorts of questions there are several missing assumptions! However, in this case, the author of the stimulus doesn't advocate reducing the number of curative hours, just increasing the number of preventative hours. That could be done through a reduction in curative hours, but it also could be through increasing the total number of teaching hours. So we can't make an assumption about reducing curative hours, since the stimulus doesn't require it.

Hope that helps!
User avatar
 sdb606
  • Posts: 78
  • Joined: Feb 22, 2021
|
#87831
The negation of E says that preventative medicine is already being taught thoroughly but just because something is already being taught thoroughly doesn't mean it can't be taught EVEN MORE thoroughly. So I still don't see how E is correct.
 Robert Carroll
PowerScore Staff
  • PowerScore Staff
  • Posts: 1787
  • Joined: Dec 06, 2013
|
#87842
Sdb,

If it were being taught thoroughly, even if it could be taught more thoroughly, wouldn't that be at least sufficient? So if the current level of teaching is thorough, then even more strongly, it's not insufficient. The conclusion is not saying it's less than perfect - it's going so far as to say it's insufficient.

Robert Carroll

Get the most out of your LSAT Prep Plus subscription.

Analyze and track your performance with our Testing and Analytics Package.